Q7

 
charmayne.palomba
Thanks Received: 24
LSAT Geek
 
Posts: 18
Joined: July 06th, 2010
 
 
 

Q7

by charmayne.palomba Wed Nov 30, 2011 4:43 pm

7. (D)
Question Type: Unconditional


If we’ve framed the game, this is a quick question. L can go last, so (A) is out.

We know O has to go last in Frame 1, so we can eliminate (B).
P can go in slot 5 in either frame, so (C) is out.

We know at least one letter, W, has to come after T. And our frames tell us that there are only two options for the last position: O or L. Therefore, the earliest position T can occupy is position 4, and (D) is the correct answer.

While we probably wouldn’t have to check (E) when actually solving this game, there is no reason W can’t go in slot 5 in either frame, so (E) is incorrect.